MacLaurin Series for f(x)=ln|1+x^3|

Click For Summary
The discussion centers on finding the MacLaurin series representation for the function f(x) = ln|1+x^3|. The user has calculated several derivatives at x=0, yielding values of 0 for the first three derivatives and 6 for the third derivative, indicating a pattern in the coefficients. They express frustration in deriving the series and mention a complex numerator that resembles a polynomial. The user also plans to determine the radius of convergence using the ratio test and to approximate the integral of ln|1+x^3|. The conversation highlights the challenges in applying the MacLaurin series and suggests using the geometric series for the derivative as a potential solution.
Forknknife
Messages
1
Reaction score
0

Homework Statement


Find the MacLaurin series representation for f(x)=ln|1+x^3|

Homework Equations


1/(1-x) = \sumx^n = 1+x+x^2+x^3+... |x|<1

The Attempt at a Solution


right.
so maclaurin series by default means it expands as a taylor series where x=0
f(0)= ln|1+x^3| = 0
f'(0)= 3(0)^2/(1+0^3)^1 = 0/1 = 0
f''(0)= -3(0)^2/(1+0^3)^2 = -0/1 = 0
f'''(0)= 6(0)^6-42(0)^3+6/(1+0^3)^3 = 6
so on and so forth

it's taken me a few hours but so far, i can't seem to use that relevant equation to find the maclaurin series.

i may have gotten kinda close, b/c I've found so far that
fn(0)=(very complex numerator that kind of looks like a quadratic or polynomial formula with switching signs)/(1+x3)n

i've done upto 9th derivative(and further more) and found that this goes something like:
0,0,0,6,0,0,-360,0,0,120960,0,0,-119750400,0,0,261534973600
and that the series the pretty much follows (-1)^(n-1)*3*(n-1)!, where n starts at 0, with 2 blanks in between.

i know I'm missing something, and this isn't that hard.
please help.

--
then I'm supposed to find radius of convergence, which i think i can probably get using by ratio test..
then need to find out how many terms are needed to appx it within 0.0001...
then find out why appx'ing integral of ln|1+x^3| from 0 to 2 by using the series representation is wrong.
but i think i can figure those out once the first part is done.
 
Last edited:
Physics news on Phys.org
Find the Maclaurin series for the derivative of ln(1+x^3), by using the geometric series.
 
Or find the Maclaurin series for ln(1+t) and then let t=x^3.
 
Question: A clock's minute hand has length 4 and its hour hand has length 3. What is the distance between the tips at the moment when it is increasing most rapidly?(Putnam Exam Question) Answer: Making assumption that both the hands moves at constant angular velocities, the answer is ## \sqrt{7} .## But don't you think this assumption is somewhat doubtful and wrong?

Similar threads

  • · Replies 48 ·
2
Replies
48
Views
5K
  • · Replies 6 ·
Replies
6
Views
2K
  • · Replies 10 ·
Replies
10
Views
2K
  • · Replies 16 ·
Replies
16
Views
3K
  • · Replies 7 ·
Replies
7
Views
2K
  • · Replies 1 ·
Replies
1
Views
2K
Replies
6
Views
2K
  • · Replies 7 ·
Replies
7
Views
2K
  • · Replies 2 ·
Replies
2
Views
5K
  • · Replies 12 ·
Replies
12
Views
2K